Đến nội dung

nam8298 nội dung

Có 158 mục bởi nam8298 (Tìm giới hạn từ 08-05-2020)



Sắp theo                Sắp xếp  

#460920 $a^3+b^3+c^3-3abc\geq 2\left ( \frac{b+c}{...

Đã gửi bởi nam8298 on 30-10-2013 - 19:59 trong Bất đẳng thức - Cực trị

+ nếu $\frac{b+c}{2}-a\leq 0$ ta đc đpcm

+nếu $\frac{b+c}{2}-a> 0$   đặt b=a+2x ; c=a+2y 

    đặt A= $a^{3}+b^{3}+c^{3}-3abc-2(\frac{b+c}{2}-a)^{3}$   suy ra A= $12a(x^{2}-xy+y^{2})+6(x+y)(x-y)^{2}\geq 6(x+y)(x-y)^{2}= \frac{3}{2}(\frac{b+c}{2}-a)(b-c)^{2}\geq 0$    suy ra BĐT đc cm




#456774 $\sum \frac{1}{c\sqrt{a^{2}...

Đã gửi bởi nam8298 on 11-10-2013 - 12:45 trong Bất đẳng thức và cực trị

cho ab+bc+ca =1.CMR $\sum \frac{1}{c\sqrt{a^{2}+b^{2}}}\geq \frac{9}{2\sqrt{3}-3\sqrt{6}abc}$




#481187 $y^{3}-1=x^{4}+x^{2}$

Đã gửi bởi nam8298 on 05-02-2014 - 19:27 trong Số học

mình có cách như thế này nhưng ko biết đúng hay ko

trước hết $x=1$ => $y=\sqrt[3]{3}$ nên loại

$y^3 = x^4 + x^2 +1 = (x^2+x+1)(x^2-x+1)$

lại có $(x^2+x+1, x^2-x+1)=1 (*)$

nên ta chia ra 2 trường hợp

$\left\{\begin{matrix} x^2-x+1=1& & \\ x^2+x+1=y^3& & \end{matrix}\right.$

$\left\{\begin{matrix} x^2+x+1=1& & \\ x^2-x+1=y^3& & \end{matrix}\right.$

sẽ suy ra nghiệm $x=0$ và $y=1$

chứng minh $(*)$
Gọi $d$ là ƯCLN$(x^2+x+1, x^2-x+1)$
=> lập hiệu $x^2+x+1- x^2+x-1 = 2x$

=> $d$ là ước của $2x$

mà ta đều có $x^2 + x +1 =x(x+1)+1$ và $x^2- x +1 =x(x-1)+1$ đều là số lẻ nên ko chia hết cho 2 và đều chia x dư 1 => không chia hết cho $2x$
=> $d=1$
 

mình nghĩ không đúng bởi vì chẳng hạn  y = p.q với p,q là 2 số nguyên tố thì cái chữ đỏ đấy ko đúng




#484446 $\sum \sqrt{a+(b-c)^{2}}\geq \sq...

Đã gửi bởi nam8298 on 23-02-2014 - 20:16 trong Bất đẳng thức - Cực trị

bài này  bình phương xong dùng Cauchy-Schwazt .

pp làm là cách nâng lũy thừa và điều chỉnh hệ số.

nó tương tự bài bđt thi chọn đôi tuyển Vĩnh Phúc năm 2013-2014.  lời giải hơi dài nên giờ mình ko kịp đánh ra




#480024 CMR: $\sum \sqrt[4]{\frac{\sqrt{3...

Đã gửi bởi nam8298 on 30-01-2014 - 11:44 trong Bất đẳng thức - Cực trị

mình có cách khác như sau BĐT cần chứng minh tương đương với $\sum \sqrt[4]{a^{3}b^{4}c^{4}(\sqrt{3}+6\sqrt{3}ab)}\leq 1$

dùng AM- GM cho 4 số 9abc ,9abc,9abc và $\sqrt{3}bc+6\sqrt{3}ab^{2}c$ thì đc BĐT cần chứng minh




#480018 Tìm GTLN của P=$\mid (x-y)(y-z)(z-x)\mid$

Đã gửi bởi nam8298 on 30-01-2014 - 11:40 trong Bất đẳng thức - Cực trị

ta có P$P^{2}= 108-27x^{2}y^{2}z^{2}\leq 108 \Rightarrow P\leq 6\sqrt{3}$

dấu bằng xảy ra khi  (x,y,z ) =( căn 3 ,- căn 3 ,o ) và các hoán vị




#485014 Cho các số thực dương x, y,z thỏa mãn x+y+z=1. Tìm Min của biểu thức: P=...

Đã gửi bởi nam8298 on 27-02-2014 - 14:16 trong Bất đẳng thức và cực trị

dùng phương pháp đổi biến p ,q, r

ta chứng minh P $\geq \frac{1}{4}$

viết $\frac{1}{4}= \frac{(x+y+z)^{3}}{4}$

nhân hết lên ta đc bđt Schur




#489249 $abcd-1\vdots (a-1)(b-1)(c-1)(d-1)$

Đã gửi bởi nam8298 on 28-03-2014 - 19:51 trong Số học

đặt a-1 =x

      b-1 =y

      c-1 =z

       d-1 =t

sau đó thay vào đk.nhân bung ra. rút gọn 1 và xyzt đc A chia hết cho xyzt. đặt A = k .xyzt

chia 2 vế cho xyzt rồi chặn.tìm đc k rồi tìm x,y,z,t




#503840 CMR: $\frac{a^{3}}{x^{2}}+...

Đã gửi bởi nam8298 on 03-06-2014 - 20:20 trong Bất đẳng thức và cực trị

Áp dụng AM-GM ta có $\frac{a^{3}}{x^{2}}+2x \geq 3a$

Tương tự cộng vế là xong




#499253 $P=\frac{a+b}{2c^2+ab}+\frac{b+c...

Đã gửi bởi nam8298 on 15-05-2014 - 20:42 trong Bất đẳng thức và cực trị

Xem tại đây




#489251 Tìm min M=$x^{3}+y^{3}+z^{3}= 12$.

Đã gửi bởi nam8298 on 28-03-2014 - 19:55 trong Bất đẳng thức và cực trị

chắcđề yêu cầu tìm min của M

xét $(x+1)(x-2)^{2}\geq 0$

nhân bung ra .

tương tự cho y và z rồi cộng vế




#477088 Tứ Giác Đều Hòa

Đã gửi bởi nam8298 on 13-01-2014 - 18:54 trong Hình học

đây là tíng chất cơ bản của tứ giác điều hòa mà bạn




#486481 $\sum \frac{x}{1+y^{3}}\geq...

Đã gửi bởi nam8298 on 12-03-2014 - 19:42 trong Bất đẳng thức - Cực trị

do x ,y ,z nguyên dương nên x=y=z =1

bđt luôn đúng




#476405 giả sử $a^2+b^2=1$ ;chứng minh $((a+b)^2-(a+b))^2\geq 4(a...

Đã gửi bởi nam8298 on 09-01-2014 - 21:19 trong Bất đẳng thức và cực trị

do $a^{2}+b^{2}=1 \Rightarrow -1 \leq a,b\leq 1$

ta có $(a+b)^{2}=2ab+1$ .do đó sau khi nhân hết ra ta đc bđt cần chứng minh tương đương với (1-a)(1-b)$\geq 0$ (luôn đúng )




#460925 Chứng minh $\sum \sqrt{1-\frac{(x+y^{2...

Đã gửi bởi nam8298 on 30-10-2013 - 20:11 trong Bất đẳng thức - Cực trị

bài này bình phương rồi dùng Cauchy-Schwazt




#457046 $3(x^{2}+xy+y^{2})(y^{2}+yz+z^{2...

Đã gửi bởi nam8298 on 12-10-2013 - 12:38 trong Bất đẳng thức và cực trị

x,y,z  là các số thực chứ có phải dương đâu mà làm thế đc81(x+y)2(y+z)2(z+x)264(x+y+z)2(xy+yz+xz)29(x+y)(y+z)(z+x)8(x+y+z)(xy+yz+zx)




#456892 Chứng minh

Đã gửi bởi nam8298 on 11-10-2013 - 20:40 trong Bất đẳng thức và cực trị

 ta có $2(a+b+c)-abc=a(2-bc)+2(b+c)\leq \sqrt{(a^{2}+(b+c)^{2})((2-bc)^{2}+4)}= \sqrt{(9+2bc)(b^{2}c^{2}-4bc+8)}$.đến đây biến đổi tương đương là đc




#456888 $3(x^{2}+xy+y^{2})(y^{2}+yz+z^{2...

Đã gửi bởi nam8298 on 11-10-2013 - 20:28 trong Bất đẳng thức và cực trị

cho x,y,z là các số thực CMR:$3(x^{2}+xy+y^{2})(y^{2}+yz+z^{2})(z^{2}+zx+x^{2})\geq (x+y+z)^{2}(xy+yz+zx)^{2}$




#460926 $\frac{bc}{3a^2+b^2+c^2}+\frac{ca...

Đã gửi bởi nam8298 on 30-10-2013 - 20:17 trong Bất đẳng thức - Cực trị

theo mình đánh giá thế này $\frac{bc}{3a^{2}+b^{2}+c^{2}}\leq \frac{(b+c)^{2}}{12a^{2}+2(b+c)^{2}}$   

chuẩn hóa a+b+c =3 .sau đó dùng ước lượng là đc




#471614 Cho 3 số thực dương a, b,c thỏa mãn a+b+c=3. CMR: $\sqrt{a...

Đã gửi bởi nam8298 on 18-12-2013 - 20:08 trong Bất đẳng thức và cực trị

bài 2  :áp dụng AM-Gm ta có $\sqrt{\frac{x}{2}}\leq \frac{2x+1}{2}$

tương tự cho y

ta chứng minh $\frac{2x+1}{1+y}+\frac{2y+1}{1+x}\leq \frac{8}{3}$

tương đương $\frac{2x^{2}+2y^{2}+3x+3y+2}{1+xy+x+y}\leq \frac{8}{3}$

ta có (x-0,5)(y-0,5)$\geq 0$ tương đương 4xy+1$\geq$ 2(x+y)

mà $2x^{2}\leq x;2y^{2}\leq y$ .từ đó biến đổi ra đpcm




#456176 $\frac{1}{4a^{2}+b^{2}+c^{2...

Đã gửi bởi nam8298 on 08-10-2013 - 20:24 trong Bất đẳng thức và cực trị

áp dụng cauchy -schwazt ta có $(4+1+1)(4a^{2}+b^{2}+c^{2})\geq (4a+b+c)^{2}$

tương tự với 2 mẫu còn lại.ta phải chứng minh $\sum \frac{1}{a^{2}+2a+1}\leq \frac{3}{4}$

đến đây chứng minh $\frac{1}{a^{2}+2a+1} \leq \frac{1}{4}+k(a-1)$    .tương tự rồi cộng vế suy ra đpcm




#472510 $A=a^3+b^3+c^3+4abc\leq \frac{9}{32}$

Đã gửi bởi nam8298 on 23-12-2013 - 19:50 trong Bất đẳng thức và cực trị

giả sử a là max {a,b,c}

BĐT cần chứng minh tương đương với $\frac{23}{32}+7abc\leq 3(ab+bc+ca)$

mặt khác ta có $(1-2a)(1-2b)(1-2c)\geq 0\Leftrightarrow 3(ab+bc+ca)\geq 6abc+\frac{3}{4}$

ta chứng minh 6abc+$6abc+\frac{3}{4}\geq 7abc+\frac{23}{32}\Leftrightarrow \frac{1}{32}\geq abc$

áp dụng AM-GM ta có abc $\leq a\frac{(b+c)^{2}}{4}= a\frac{(1-a)^{2}}{4}\leq \frac{1}{32}$




#471633 $\sqrt{2(4x^{3}-1)}-2x=\sqrt[3]{4x^...

Đã gửi bởi nam8298 on 18-12-2013 - 20:41 trong Phương trình - Hệ phương trình - Bất phương trình

 giải phương trình sau $\sqrt{2(4x^{3}-1)}-2x=\sqrt[3]{4x^{2}-1}$




#471627 BĐT $\frac{6(a+b+c)(ab+bc+ca)}{(a+b)(b+c)(c+a)}...

Đã gửi bởi nam8298 on 18-12-2013 - 20:34 trong Bất đẳng thức - Cực trị

1.a,b,c >0 .CMR $\frac{6(a+b+c)(ab+bc+ca)}{(a+b)(b+c)(c+a)}+\frac{(a+b)^{2}(b+c)^{2}(c+a)^{2}}{abc(a+b+c)^{3}}\geq \frac{985}{108}$

2.x,y,z > 0 thỏa mãn xy+yz+zx =1.Tìm min  $\sum \frac{x^{2}}{1+x(x+\sqrt{1+x^{2}})}$




#476473 Từ các số 1,2,3,4,5,6,7, 8,9 người ta lập tất cả các số có 4 chữ s...

Đã gửi bởi nam8298 on 10-01-2014 - 12:18 trong Tổ hợp - Xác suất và thống kê - Số phức

tinh số lần xuất hiện của mỗi chữ số

 

Buớc tính tổng là sao mình ko biết?